Đến nội dung

thinhrost1

thinhrost1

Đăng ký: 28-04-2013
Offline Đăng nhập: 07-06-2019 - 12:37
****-

#663295 Olympic Trường đông tóan học miền nam 2016

Gửi bởi thinhrost1 trong 28-11-2016 - 18:57

Ngày 1 (28-11)
Nguồn: Official fanpage of Trường đông
attachicon.gif15178232_1183896728358169_8722552571728938551_n.png.jpg

Câu 4:

a. Gọi $a_i$ là số ô đen ở hàng thứ $i (i=1,2,....,2n)$

           $b $là số ô đen ở mỗi cột.

Ta có$ a_1+a_2+....+a_{2n}=2nb$

$a_1, a_2,.....,a_{2n}$ là $2n$ số nguyên phân biệt nằm thuộc [0;2n]

=> $a_1+a_2+....+a_{2n=}1+2+....+2n-j=n(2n+1)-j$ (với $j$ là số nguyên thuộc [0;2n] và không phải là một trong các số $a_i$)

=>$n(2n+1)-j$ chia hết cho $2n$

=>$j=n$

Vậy tất cả có $2n^2$ ô đen.

b. - Ta đánh giá số cặp khác màu trên hàng:

Mỗi hàng có a ô đen.

Trường hợp 1: $a < n $ta có số cặp tối đa là $2a$ cặp.

Trường hợp 2: $a \ge n$ ta có số cặp tối đa là $2(2n-a)$ cặp.

Vậy Tổng số cặp khác màu trên hàng $\le 4(1+2+...+n-1)=2n(n-1)$.

- Ta đánh giá số cặp khác màu theo cột:

Với hàng $i (i \ge 2)$ bất kì tồn tại ô cùng màu với ô kề ở hàng trên hoặc hàng dưới của nó.

Nên số cặp khác màu trên cột $ \le (2n-1)2n-(n-1)$.

Vậy số cặp khác màu lớn nhất có thế là $6n^2-5n+1$.

Việc cuối cùng chỉ còn chỉ ra dấu bằng.

 

Bài 3 chỉ là định lý con bướm trong tam giác.




#662416 $ 2^k| n^n-m$

Gửi bởi thinhrost1 trong 19-11-2016 - 15:33

Ta sẽ chứng minh quy nạp theo $ k$.

 

Trường hợp $ k=1$ là hiển nhiên, chọn $ n=1$.

Giả sử rằng tồn tại $ n$ sao cho $ 2^k|n^n-m$,Đặt $ n=n_0$ hay $ 2^k|n_0^{n_0}-m$ hiển nhiên $ n_0$ lẻ

 

Tiếp theo xét 2 trường hợp

 

$ 1)$ Nếu $ 2^{k+1} | n_0^{n_0}-m$

Rõ ràng chỉ cần chọn $ n=n_0$.

 

$ 2)$ Nếu $ 2^{k+1}$ Không là ước của $ n_0^{n_0}-m$.

Đặt $ n_0^{n_0}-m=u\cdot 2^k$ Với $ u$ lẻ và theo định lý Euler,Cho mọi số lẻ $ a$ ta có $ a^{2^{k}}\equiv 1\pmod{2^{k+1}}$,Vì thế

$ (2^k+n_0)^{2^k+n_0}-m=(2^k+n_0)^{2^k}\cdot (2^k+n_0)^{n_0}-m\equiv (2^k+n_0)^{n_0}-m\\\equiv n_0^{n_0}+n_0\cdot n_0^{n_0-1}\cdot 2^k-m=(u+n_0^{n_0})2^k\equiv 0\pmod{2^{k+1}}$ Vì cả $ u$ $ n_0$ đều lẻ.

 

Vì thế $ n=2^k+n_0$ thỏa mãn, nên ta có đpcm




#661981 Chứng minh $\sum\frac{1+x^{2}}{1+y+z...

Gửi bởi thinhrost1 trong 15-11-2016 - 06:16

Bài 1(Marian Tetiva):  cho $x,y,z>0$ và $x+y+z=xyz$ 

chứng minh$xy+yz+zx\geq 3+ \sum (\sqrt{x^{2}+1})$

 

 

Bài 2 (JBMO 2003): cho $x,y,z > -1$

chứng minh $\sum\frac{1+x^{2}}{1+y+z^{2}}\geq 2$

 

 

Bài 3 (Russia MO 2002): cho $x,y,z>0$ và $x+y+z=3$ 

chứng minh: $\sqrt{x} + \sqrt{y} + \sqrt{z} \geq xy+yz+xz$

Bài 3: Câu dễ nhất trước:

 

Bất đẳng thức cần chứng minh tương đương:

$ \sum 2\sqrt{x} \ge 2 \sum xy \\\Leftrightarrow \sum (x^2+2\sqrt{x}) \ge 9 $

 

Lưu ý theo AM-GM:

$x^2 +\sqrt{x} +\sqrt{x} \ge 3\sqrt[3]{x^3}=3x$




#661960 $\frac{2x}{\sqrt{1+x^{2}}...

Gửi bởi thinhrost1 trong 14-11-2016 - 22:28

1:Cho 3 số dương x,y,z thỏa mãn xy+yz+xz=1, chứng minh:

$\frac{2x}{\sqrt{1+x^{2}}}+\frac{y}{\sqrt{1+y^2}}+\frac{z}{\sqrt{1+z^2}}$$\leq \frac{9}{4}$

(ưu tiên cách dùng lượng giác :D )

Đặt $x=tan\frac{A}{2},y=tan\frac{B}{2},z=tan\frac{C}{2}$ A,B,C là 3 góc của tam giác. Thay vào ta chỉ cần chứng minh:

$2sin\frac{A}{2}+sin\frac{B}{2}+sin\frac{C}{2}=2cos(\frac{B+C}{2})+2sin(\frac{B+C}{4})cos(\frac{B-C}{4})\le 2(1-2sin^2\frac{B+C}{4})+2sin(\frac{B+C}{4})=-4(sin\frac{B+C}{4}-\frac{1}{4})^2+\frac{9}{4} \le \frac{9}{4}$




#661846 Kỳ thi chọn đội tuyển dự thi VMO tỉnh Đồng Nai

Gửi bởi thinhrost1 trong 13-11-2016 - 23:41

Bài hình có cách giải khá đơn giản như sau:

Gọi M là trung điểm BC.

$(ADE), (BCQE), (BDPC)$ có 3 trục đẳng phương là $DP, QE, BC$ đồng quy.

 

Áp dụng định lý Pascal đảo do $S, B, C$ thẳng hàng nên $BQ, CP$ cắt nhau tại $R$ trên $(ADE)$

 

$ \angle ARB =180^o- \angle AEP=180^o- \angle ADP= 180^o- \angle RCB$ Như vậy $AR$ song song $BC$.

 

Mặt khác nếu $R'$ là điểm đối xứng với M qua $DE$ CM được $R'$ thuộc (AED) và $AR'$ song song $BC$. Vậy $R$ trùng $R'$ ta có đpcm

 

ps: Bài hình có xuất hiện trước đó trong Germany VAIMO 2015




#661776 $1+2^{n}+4^{n}$ là số nguyên tố

Gửi bởi thinhrost1 trong 13-11-2016 - 16:16

Bài 1: Tìm $p,q$ nguyên tố sao cho $2^{p}+2^{q}\vdots (pq).$

Bài 2: Giả sử $1+2^{n}+4^{n}$ là số nguyên tố, $n$ nguyên dương. Tìm $n$ thỏa mãn bài toán.

Bài 1: $2^{p}+2^{q}\vdots (pq). (*)$

Trường hợp 1: $p=q$

 

Thay vào (*) ta được $p^2 | 2.2^p$ suy ra $p=2$.

 

Trường hợp 2: $ p \ne q$.

Trường hợp 2.1: Một trong hai số $p,q$ bằng 2. Không mất tính tổng quát giả sử $q=2, p \ne 2$ Khi đó $2p |2^2+2^p \Rightarrow 2^2+2^p \equiv 0 (mod p)$ Mặt khác theo định lý Fermat, ta có:

$2^p \equiv 2 (mod p)$ $\Rightarrow 0 \equiv 2^2+2=2.3 (mod p)$ nên $p=3$. Kiểm tra trực tiếp thấy thỏa mãn vậy $(2,3), (3,2)$ thỏa mãn bài toán.

Trường hợp 2.2: $p \ne q, p,q \ne 2$. Theo định lí Fermat ta có:

$0 \equiv 2^p+2^q \equiv 2+2^q \equiv 2(1+2^{q-1}) (mod p) \Rightarrow p | 2(2^{q-1}+1) \Rightarrow p | 1+ 2^{q-1} (1) \Rightarrow 2^{2(q-1)} \equiv 1 (mod p)$

 

$\Rightarrow ord_p(2)| 2(q-1) \Rightarrow ord_p(2) | (2(q-1),p-1)=2(q-1,p-1)$. Do $p,q$ là hai số nguyên tố lẻ đặt:

 

$p-1=2^lm,q-1=2^kn$ trong đó $m, n$ là các số lẻ $k, l >0$. Khi đó $ord_p(2) | 2^{k+1}$ Như vậy $ord_p(2)=2^h ( 0 \le h \le k+1)$

 

Nếu $ 0 \le h \le k$ thì:  $((2^{2^h})^{2^{k-h}})^m \equiv 1 (mod p)$ hay $2^{q-1}-1 \equiv 0 (mod p)$ Mâu thuẫn với (1).

 

Vậy $ord_p(2)=2^{k+1} \Rightarrow 2^{k+1} | \varphi (p)=p-1=2^lm \Rightarrow 2^{k+1}|2^l \Rightarrow k<l$. Tương tự ta chứng minh được $l < k$ (vô lí). 

 

Kết luận $(p,q)=(2,2), (2,3), (3,2)$ thỏa mãn bài toán

 

Bài tương tự: Tìm $p,q$ nguyên tố sao cho $5^p+5^q \vdots pq$

Bài 2: bạn có thể xem lại đề không mình chỉ chứng minh được $n$ có dạng $3^k$ :(




#661226 ĐỀ THI HSG THPT CHUYÊN VÀ CHỌN ĐỘI TUYỂN DỰ THI HSG QUỐC GIA THPT NĂM 2016-20...

Gửi bởi thinhrost1 trong 09-11-2016 - 09:49

 

 

Câu 5. (3 điểm) Tìm tất cả các đa thức $ P(x) $ với hệ số thực thỏa mãn điều kiện:

$ P(x^2)+P(x).P(x+1)=0 $ với mọi $ x \in \mathbb{R} $

 

 

Chém nốt câu 5:

 

Do $ P(x^2)=-P(x).P(x+1)$ với mọi $ x \in \mathbb{R} $

 

Suy ra: $ P(x^2)=-P(x).P(x+1)$ với mọi $ x$

 

Nếu $P(x)$ là đa thức hằng thì $P(x) \equiv 0$ hoặc -1

Nếu $P(x)$ không phải đa thức hằng:

 

Giả sử $\alpha$ là nghiệm của $P(x)=0$.

 

Khi đó từ phương trình suy ra $\alpha ^2, \alpha^4, \alpha ^8,..$ cũng là nghiệm của $P(x)=0$. Từ đây suy ra $|\alpha|=0$ hoặc $| \alpha |=1$, vì nếu ngược lại sẽ thu được dãy vô hạn các nghiệm của P(x). Tương tự $\alpha -1$ cũng là nghiệm của $P(x)$ và lý luận tương tự, ta cũng dc $| \alpha-1|=0$ hoặc $| \alpha -1|=1$

 

Giả sử rằng $|\alpha |=1$ và $|\alpha -1|=1$. Ta viết $ \alpha = cos \varnothing  +isin \varnothing $ Ta thấy rằng $2 cos \varnothing =1$ Từ đây suy ra: $\varnothing = \frac{\pi}{3}$ hoặc $\varnothing = \frac{5 \pi}{3}$ Ta xết các trường hợp:

Giả sử $\varnothing = \frac{\pi}{3}$ Xét $ \alpha ^2$ cũng là nghiệm của $P(x)=0$. Như vậy $ \alpha ^2 -1$ cũng là nghiệm của $P(x)=0$ và $| \alpha ^2-1|=\left ( cos\frac{2\pi}{3}-1 \right )^2+sin^2\left ( \frac{2 \pi}{3} \right )=3$ Mâu thuẫn vì mọi nghiệm của $P(x)=0$ đều có module bằng 0 hoặc 1. Tương tự với $\varnothing = \frac{5 \pi}{3}$.

 

Như vậy ta có thể kết luận rằng $ \alpha =1$ hay $ \alpha -1=0$. Từ đây P(x) có dạng $cx^m(1-x)^n$ với $c$ là một số nào đó và $m, n$ là các số nguyên không

âm

 

Thay vào phương trình:

 

Ta được $m=n=2k, c=-1$. Thử lại thấy thỏa mãn.

 

Vậy: nghiệm của phương trình hàm là: $P(x) \equiv 0$

$P(x) \equiv -1$

$P(x) \equiv -x^{2k}(1-x)^{2k}$ ( $k$ là số nguyên không âm bất kì)




#661223 ĐỀ THI HSG THPT CHUYÊN VÀ CHỌN ĐỘI TUYỂN DỰ THI HSG QUỐC GIA THPT NĂM 2016-20...

Gửi bởi thinhrost1 trong 09-11-2016 - 09:04

Câu 7: 

Đặt $d=t(a+b+c),t>0$
 
$(a+b+c+d)^3 \leq 4(a^3+b^3+c^3+d^3)+24\bigl( (ab(c+d)+cd(a+b) \bigr)$
 
$ \Longleftrightarrow \sum_{cyc} S_c(a-b)^2+9abc(1+15t+9t^3-9t^2)\geq 0$.
 
 
Ta chỉ cần chứng minh   $\sum_{cyc} S_c(a-b)^2\geq 0.$
Không mất tính tổng quát giả sử  $a\geq b \geq c$
 
$(1)S_b=(3-3t-3t^2+3t^3)a+(3-3t-3t^2+3t^3)c+(-3-21t^2+21t^3+27t)b$
 
$\geq [(3-3t-3t^2+3t^3)+$ $(-3-21t^2+21t^3+27t)]b+(3-3t-3t^2+3t^3)c$ 
 
$=(24t-24t^2+24t^3)b+(3-3t-3t^2+3t^3)c\geq 0$
 
 
$(2)S_b+S_c\geq S_c=(3-3t-3t^2+3t^3)b+(3-3t-3t^2+3t^3)a+(-3-21t^2+21t^3+27t)c$
 
$\geq 0+(3-3t-3t^2+3t^3)c+(-3-21t^2+21t^3+27t)c= (24t-24t^2+24t^3)c\geq 0$
 
 
$(3)S_b+S_a=(6-6t^2-6t+6t^3)c+(24t-24t^2+24t^3)a+(24t-24t^2+24t^3)b\geq 0.$
 
Vậy : Ta có đpcm



#661102 ĐỀ THI CHỌN ĐT QG TỈNH HÀ NAM NĂM 2016-2017

Gửi bởi thinhrost1 trong 08-11-2016 - 12:50

 

Bài $4$: Cho $P,Q,R$ là $3$ đa thức hệ số thực thỏa mãn: $P(Q(x))+P(R(x))=c$ $\forall x\in\mathbb{R}$ với $c=const\in\mathbb{R}$

CMR: $P(x)\equiv const$ hoặc $[Q(x)+R(x)]\equiv const$

 

Đặt $deg P(x)=p$ và không mất tính tổng quát giả sử $deg Q(x)=q \ge r=degR(x)$. Nếu $P(x)\equiv const$ hoặc $Q(x)\equiv const$ thì khi đó $R(x)\equiv const$ nên hai trường hợp này là hiển nhiên. Ta xét $ p,q >0$, $r \ge 0$

Đặt $C_k (f(x))$ là hệ số của $x^k$ trong đa thức  $f(x)$, vì thế $C_{\deg f(x)} (f(x)) \neq 0$ là hệ số cao nhất của $f(x)$. Đặt $a = C_{\deg P(x)} (P(x))$, $b = C_{\deg Q(x)} (Q(x))$, $c = C_{\deg R(x)} (R(x))$.

Nếu $q>r$, Khi đó $C_{pq} (P(Q(x)) + P(R(x))) = ab^p \neq 0$, Vô lí. Vì thế ta phải có $q=r=m$, $\Rightarrow$ $C_{pm} (P(Q(x)) + P(R(x))) = a(b^p +c^p)\neq 0$, Theo điều kiện giả thiết thì $b^p + c^p = 0$, dẫn tới $p$ lẻ và $c=-b$.
 
Xét $a(Q(x)^p + R(x)^p) = a(Q(x)+R(x)) S(x)$, Trong đó $ S(x) = Q(x)^{p-1} - Q(x)^{p-2}R(x) + \cdots - Q(x)R(x)^{p-2} + R(x)^{p-1}$. Ta có $C_{(p-1)m}(S(x)) = b^{p-1} - b^{p-2}(-b) + \cdots - b(-b)^{p-2} + (-b)^{p-1} = pb^{p-1} \neq 0$, nên $\deg S(x) \geq (p-1)m$ (Thực ra là bằng luôn). 
 
Mặt khác  nếu đặt $T(x) = P(Q(x)) + P(R(x)) - a(Q(x)^p + R(x)^p)$ ta có $\deg T(x) \leq (p-1)m$. Giả sử ngược lại $Q(x)+R(x)$ không là hằng số, $\Rightarrow$ $\deg(Q(x)+R(x)) \geq 1$, Ta phải có $\deg(a(Q(x)^p + R(x)^p)) = \deg(a(Q(x)+R(x)) S(x)) \geq 1 + (p-1)m$, và vì thế $\deg(P(Q(x)) + P(R(x))) = \deg(a(Q(x)^p + R(x)^p) + T(x)) \geq 1+(p-1)m >0$, Mâu thuẫn.
 
Vậy ta có đpcm.
Ps: Nếu tồn tại $Q,R$ mà $Q(x) + R(x) = C$ là hằng số, ta vẫn có thể tìm đa thức $P$ khác hằng bậc$ p$ lẻ bất kì, để $P(Q(x)) + P(R(x)) $ là hằng số. Chỉ cần lấy $P(x) = (2x-C)^p + k/2$



#660971 ĐỀ THI CHỌN ĐT QG TỈNH LẠNG SƠN NĂM 2016-2017

Gửi bởi thinhrost1 trong 07-11-2016 - 16:36

Bài 4 : Ta chứng minh tồn tại $a,b$ để $4x^3-18x^2+27x+m=P(x) \equiv 4(x+a)^3+b=4x^3+12ax^2+12a^2x+b+4a^3 \pmod{107}$  
Chọn $a \in \mathbb{Z}$ sao cho $12a \equiv -18 \equiv 89 \pmod{107} \Rightarrow a \equiv 52 \pmod{107},(12,107)=1 \Rightarrow 12.52^2 \equiv 27 \pmod{107}$ 
Vậy với mọi $m$ ,chọn $b$ sao cho $b \equiv m-4a^3 \pmod{107}$ 

 

Có thể thay $107$ bởi $p=3k+2$ bất kì (p nguyên tố).

 

https://toanhocsocap...o-dang-3k2.html




#660966 Trường hè toán học năm 2016 (phần đại số)

Gửi bởi thinhrost1 trong 07-11-2016 - 15:14

 

 

37. Tìm tất cả các số thực $k$ sao cho tồn tại các số thực dương $a,b,c$ thỏa mãn $abc=1$ và $\frac{1}{{{a}^{k}}(b+c)}+\frac{1}{{{b}^{k}}(c+a)}+\frac{1}{{{c}^{k}}(a+b)}<\frac{3}{2}.$

 

- Xét trường hợp $k \ge 2$. Ta sẽ chứng minh $\frac{1}{{{a}^{k}}(b+c)}+\frac{1}{{{b}^{k}}(c+a)}+\frac{1}{{{c}^{k}}(a+b)} \ge \frac{3}{2}.$ với mọi $a,b,c$ dương thỏa mãn $abc=1$. Không mất tính tổng quát giả sử $ a \le b \le c$. Đặt: $x=\frac{1}{a}, y=\frac{1}{b},z=\frac{1}{c}$. Khi đó $x,y,z >0$ và $xyz=1$. Bất đẳng thức cần chứng minh trở thành:

$\sum \frac{x^{k-1}}{y+z} \ge \frac{3}{2}$

Ta có:$x \geq y\geq z \Rightarrow \frac{x}{y+z} \geq \frac{y}{z+x} \geq \frac{z}{x+y}$

Theo bất đẳng thức Chebyshev :

$(x^{k-2}+y^{k-2}+z^{k-2})(\frac{x}{y+z}+\frac{y}{z+x}+\frac{z}{x+y}) \le 3(\frac{x^{k-1}}{y+z}+\frac{y^{k-1}}{z+x}+\frac{z^{k-1}}{x+y})$.

Mà theo Nesbitt: $\frac{x}{y+z}+\frac{y}{z+x}+\frac{z}{x+y} \ge \frac{3}{2}$, và AM-GM: $x^{k-2}+y^{k-2}+z^{k-2} \ge 3$ nên:

$\frac{x^{k-1}}{y+z}+\frac{y^{k-1}}{z+x}+\frac{z^{k-1}}{x+y} \geq \frac{3}{2}$

Vậy $k \ge 2$ không thỏa mãn.

- Xét trường hợp $ \frac{1}{2}<k<2$ Cho $a=b=\frac{1}{n}$ $c=n^2$. Thì: 

$S=\frac{1}{{{a}^{k}}(b+c)}+\frac{1}{{{b}^{k}}(c+a)}+\frac{1}{{{c}^{k}}(a+b)}=\frac{2n^{k+1}}{n^3+1}+\frac{n^{1-2k}}{2}$

 

Khi đó $\underset{n \rightarrow  +\infty }{lim}S=0$, suy ra với $n$ đủ lớn thì $S  <\frac{3}{2}$ 

Vậy: $ \frac{1}{2}<k<2$ thỏa mãn.

- Xét trường hợp: $ \frac{1}{2}=k$. Cho $a=b=n$ $c=\frac{1}{n^2}$ Khi đó:

$S=\frac{2n^{2-\frac{1}{2}}}{n^3+1}+\frac{1}{2}$

Khi đó $\underset{n \rightarrow  +\infty }{lim}S=\frac{1}{2}$, suy ra với $n$ đủ lớn thì $S  <\frac{3}{2}$ 

Vậy $ \frac{1}{2}=k$ thỏa mãn.

- Xét trường hợp $-1 <k < \frac{1}{2}$. Chọn $a=b=n$ $c=\frac{1}{n^2}$ thì:

$S=\frac{2n^{2-k}}{n^3+1}+\frac{n^{2k-1}}{2} \Rightarrow \underset{n \rightarrow  +\infty }{lim}S=0$

Nên  $-1 <k < \frac{1}{2}$ thỏa mãn.

- Xét trường hợp $ k \le -1$. Ta chứng minh: $\frac{1}{{{a}^{k}}(b+c)}+\frac{1}{{{b}^{k}}(c+a)}+\frac{1}{{{c}^{k}}(a+b)} \ge \frac{3}{2}.$

Nếu đặt $h=1-k \ge 2$ thì theo chứng minh trên:

$\frac{1}{{{x}^{h}}(y+z)}+\frac{1}{{{y}^{h}}(z+x)}+\frac{1}{{{z}^{h}}(x+y)} \ge \frac{3}{2}\\\Leftrightarrow \frac{a^{h-1}}{b+c}+\frac{b^{h-1}}{c+a}+\frac{c^{h-1}}{a+b}  \geq \frac{3}{2} \\\Leftrightarrow \frac{a^{-k}}{b+c}+\frac{b^{-k}}{c+a}+\frac{c^{-k}}{a+b}  \geq \frac{3}{2} \\ \Leftrightarrow \frac{1}{{{a}^{k}}(b+c)}+\frac{1}{{{b}^{k}}(c+a)}+\frac{1}{{{c}^{k}}(a+b)} \ge \frac{3}{2}$

Vậy $ k \le -1$ không thỏa mãn điều kiện đề bài.

 

Kết luận: Tập giá trị $k$ cần tìm là: $\left ( -1;2 \right )$




#660906 ĐỀ THI HSG LỚP 12 TỈNH BÌNH ĐỊNH NĂM 2016-2017

Gửi bởi thinhrost1 trong 06-11-2016 - 22:21

Câu pth là AHSME 1979

 

Đặt $g(x)=f(x)-\frac{x^2}2+1$

Ta được

$g(x+y)=g(x)+g(y)$

 

Đây là phương trình hàm Cauchy được $g(n)=an ( n \in N)$

 

Suy ra f(x).




#660731 Đề thi chọn đội tuyển chính thức học sinh giỏi dự thi quốc gia năm 2016-2017...

Gửi bởi thinhrost1 trong 05-11-2016 - 22:44

 

Câu 3 (4,0 điểm)

 

Giả sử $q$ là một số nguyên tố , dãy $(u_n)$ được xây dựng như sau:

 

$$\left\{\begin{matrix} u_0=0 & & & \\ u_1=1 & & & \\ u_n=2u_{n-1}-qu_{n-2}\forall n\geq 2,n \in \mathbb{N} & & & \end{matrix}\right.$$

 

Tìm $q$ , biết tồn tại số tự nhiên $k$ để $u_{3k}=-3$

 

 

Hết

Giả sử $p,q$ là hai số nguyên tố , dãy $(u_n)$ được xây dựng như sau:

 

$$\left\{\begin{matrix} u_0=0 & & & \\ u_1=1 & & & \\ u_n=pu_{n-1}-qu_{n-2}\forall n\geq 2,n \in \mathbb{N} & & & \end{matrix}\right.$$

 

Tìm tất cả $p,q$ biết tồn tại số tự nhiên $k$ để $u_{3k}=-3$

 

https://toanhocsocap...-day-tuyen.html




#660558 Đề thi chọn đội tuyển chính thức học sinh giỏi dự thi quốc gia năm 2016-2017...

Gửi bởi thinhrost1 trong 04-11-2016 - 14:04

Theo bạn thì tại sao nó lại như thế giải thích giúp

Xét bài toán sau: Cho bao nhiêu hoán vị của các số {1,2;..n} sao cho có ít nhất 1 phần tử giữ nguyên vị trí ban đầu.

Gọi S là tập hợp các hoán vị của {1,2,3;..n} và $A_i$ là tập hợp các hoán vị {$a_1,a_2,..a_n$} của {1;2;3;..;n} thỏa điều kiện $a_i=i$ với $i=1,2;..n$.

Khi đó ta có:

|S|=n!

$|A_i|=(n-1)!$

 

$|A_i \cap A_j|=(n-2)!\\ .......;\\ |A_{i_1} \cap A_{i_2} \cap A_{i_3} \cap ... \cap A_{i_k}|=(n-k)! ( 1 \le i_1 <i_2<..<i_k<n)$

Theo nguyên lí bù trừ, ta có:

$|A_1 \cup A_2 \cup .. \cup A_n|=\\ \sum_{i=1}^{n}|A_i|-\sum_{1 \le i < j \le n}(A_i \cap A_j \cap A_k)-..+(-1)^n|A_1 \cap A_2 \cap A_3 ..\cap A_n| \\=C^1_n(n-1)!-C^2_n(n-2)!+..+(-1)^nC^n_n= \\n!(\frac{1}{1!}-\frac{1}{2!}+\frac{1}{3!}-+..(-1)^n.\frac{1}{n!})$ 
 
Đây là điều cần cm



#660363 $P(x)\neq 0: P(x^2-2x)=[P(x-2)]^2,\forall x\in \math...

Gửi bởi thinhrost1 trong 02-11-2016 - 20:24

Tìm tất cả các đa thức khác không $P(x)$ thỏa:

$P(x^2-2x)=[P(x-2)]^2,\forall x\in \mathbb{R}$

Thay $x$ bởi $x+1$:

 

$P(x^2-1)=P(x-1)^2$
 
Đặt $P(x)=Q(x+1)$, được $Q(x^2)=Q(x)^2$. Đến đây thì miễn bàn rồi !
 
Kết luận: $P(x) \equiv 0, P(x) \equiv (x+1)^n$